悠闲数学娱乐论坛(第2版)'s Archiver

APPSYZY 发表于 2019-4-18 01:54

$\mod$递推

已知常数$i,j,k$为正整数,设非负整数数列$\{a_n\}$满足$a_1<k$,且$a_{n+1}=(ia_n+j) \mod  k$. 求证:存在正整数$m>1$使得$a_m=a_1$.

战巡 发表于 2019-4-18 02:19

[b]回复 [url=http://kuing.orzweb.net/redirect.php?goto=findpost&pid=30883&ptid=6029]1#[/url] [i]APPSYZY[/i] [/b]

感觉有问题
假设$i=j=k$,那么$a_{n+1}=0$对任意$n>0$成立,那只要$a_1$不为零,这就不对了
或者更一般的情况,$i \mod k=0$,$j$随意,那么$a_{n+1}=j \mod k$恒成立,只要$a_1$不为这个值,还是不对

APPSYZY 发表于 2019-4-18 08:18

[i=s] 本帖最后由 APPSYZY 于 2019-4-18 08:20 编辑 [/i]

[b]回复 [url=http://kuing.orzweb.net/redirect.php?goto=findpost&pid=30885&ptid=6029]2#[/url] [i]战巡[/i] [/b]
抱歉,这道题我描述错误了,改正后是这样的:
已知常数$a,i,j,k$为正整数,设非负整数数列$\{a_n\}$满足$a_1=(ia+j) \mod  k$,且$a_{n+1}=(ia_n+j) \mod  k$. 求证:存在正整数$m>1$使得$a_m=a_1$.

realnumber 发表于 2019-4-18 15:01

[b]回复 [url=http://kuing.orzweb.net/redirect.php?goto=findpost&pid=30888&ptid=6029]3#[/url] [i]APPSYZY[/i] [/b]


    描述错误是什么意思啊,题目自己想的?
反例还是有
i=3,k=27,a=j=1
如此a1=4,a2=a3=a4=...=13

页: [1]

Powered by Discuz! Archiver 7.2  © 2001-2009 Comsenz Inc.